Here are two sums
2/7 + 1/4 5/6 - 1/4
A В
Which of the two sums is closer in value to 1/2
You must show your working and state clearly whether the answer is A or b

Answers

Answer 1

Answer:

Step-by-step explanation:

[tex]\frac{2}{7}+\frac{1}{4}[/tex]

LCD = 28

[tex]\frac{2}{7}+\frac{1}{4}=\frac{2*4+1*7}{28}\\\\=\frac{8+7}{28}\\\\=\frac{15}{28}\\[/tex]

= 0.535  {Round off to tenth}

= 0.5

[tex]\frac{5}{6}+\frac{1}{4}=\frac{5*4-1*6}{24}\\\\=\frac{20-6}{24}\\\\=\frac{14}{24}\\\\=\frac{7}{12}\\[/tex]

= 0.583 {Round off to tenth}

= 0.6

So 2/7 +1/4 is closer to 1/2


Related Questions

Write a formula for quadratic function if its graph has the vertex at point (0,6) and passes through the point (−1,−2).

Answers

Answer:

y = - 8x² + 6

Step-by-step explanation:

The equation of a parabola in vertex form is

y = a(x - h)² + k

where (h, k) are the coordinates of the vertex and a is a multiplier

Here (h, k) = (0, 6), thus

y = a(x - 0)² + 6, that is

y = ax² + 6

To find a substitute (- 1, - 2) into the equation

- 2 = a(- 1)² + 6, that is

- 2 = a + 6 ( subtract 6 from both sides )

- 8 = a

y = - 8x² + 6

The formula of the quadratic function is [tex]y = -8x^2 + 6[/tex]

A quadratic function is represented as:

[tex]y = a(x - h)^2 + k[/tex]

The vertex of the quadratic function is given as:

[tex](h,k) = (0,6)[/tex]

So, we have:

[tex]y = a(x - 0)^2 + 6[/tex]

This gives:

[tex]y = ax^2 + 6[/tex]

The quadratic function passes through (-1,-2).

So, we have:

[tex]-2 = a(-1)^2 + 6[/tex]

[tex]-2 = a + 6[/tex]

Subtract 6 from both sides

[tex]a = -8[/tex]

Substitute -8 for a in [tex]y = ax^2 + 6[/tex]

[tex]y = -8x^2 + 6[/tex]

Hence, the formula of the quadratic function is [tex]y = -8x^2 + 6[/tex]

Read more about quadratic functions at:

https://brainly.com/question/11631534

help me plz ill mark u brailiest

Answers

Answer:

The product of 21 and 18

Step-by-step explanation:

When you multiply the word is product

The product of 21 and 18

Answer:

the answer is H

Step-by-step explanation:

find the length of the unknown side round your answer to the nearest 10th

Answers

Answer:

4.4 ft

Step-by-step explanation:

The sides of a right triangle are "tied together" mathematically by an equation called the Pythagorean theorem, which says: in any right triangle, the relationship between the two sides that make the right angle (we'll call them a and b) and the side across from it (c) is

[tex]a^2+b^2=c^2[/tex]

Let's sustitute these for the values we have in our problem (measured in feet)

[tex]9^2+a^2=10^2\\81+a^2=100\\a^2=19\\a=\sqrt{19}[/tex]

19 is between the perfect squares [tex]4^2=16[/tex] and [tex]5^2=25[/tex], so [tex]\sqrt{19}[/tex] must be between 4 and 5, a little bit closer to 4. Punching the expression into the calculator confirms this with a value of [tex]\sqrt{19}\approx4.3588[/tex] which we can round to 4.4 ft.

Find the distance between the pair of points
(7, -2) and (11,-2)
(-5,2) and (-5,4)
(8,-10) and (5, -10)

Pls answer fast I REALLY NEED THE ANSWERS :)

Answers

4 units, 2 units, and 3 units (I believe)
4 units, 2 units, and 3 units :)

5 + w + 21
What is the answer helped me please

Answers

Answer:

w+5=7

Step-by-step explanation:

What is the area of the triangle shown below?

Answers

Answer:

48

Step-by-step explanation:

Split it into 2 right triangles flip one around and you have a rectangle who's area is equal to 10x6.

a=6

c=2

b=?

Solve for b:

f(1) = 1
f(2) = 2
f(n) = f(n − 2) + f(n - 1)
f(3) =?

Answers

Answer:

3

Step-by-step explanation:

To find f(3) we just need to plug in 3 wherever we see an n

f(3) = f(3 - 2) + f(3 - 1)

f(3) = f(1) + f(2)

Since f(1) and f(2) are given, we can plug in the respective values

f(3) = f(1) + f(2) → f(3) = 1 + 2 = 3

Answer:

3

Step-by-step explanation:

I just did it on Khan

32pts and BRAINLIEST FOR BST ANSWER!!
Use sigma notation to represent the sum of the first seven terms of the following sequence: −4, −6, −8, …

the summation from n equals one to 7 of quantity negative 2 minus 2 times n
the summation from n equals one to 7 of quantity negative 4 plus 2 times n
the summation from n equals one to 7 of quantity negative 2 minus 4 times n
the summation from n equals one to 7 of quantity negative 2 plus 2 times n

Answers

Answer:

B

Step-by-step explanation:

B cause

Answer:The Correct answer is C) the summation from n equals one to 6 of quantity negative 7 minus 3 times n

Step-by-step explanation:

The given Arithmetic sequence: −10, −13, −16, …

Theory : For arithmetic sequence : a, a+d,a+2d......a+(n-1)d

Formula for summation is given as Sn =

Where, a=  First term of sequence = (-10) and d= difference of any two consecutive terms = (a+d)-(a) = (-13)-(-10)= (-3)

Putting values in Summation formula

Sn =

Sn =

Sn =

Sn =

Thus, Correct answer is C) the summation from n equals one to 6 of quantity negative 7 minus 3 times n

What if the diameter of the inner sphere is 1.8 meters. What is the volume of the inflated space?

Answers

Answer:

Volume is 3.05m³

Step-by-step explanation:

This problem bothers on the mensuration of solid shapes, sphere

Step one

The formula for the volume of a sphere is expressed as

V=4/3πr³

Step two

Given data

Diameter d = 1.8m

Radius r = d/2= 1.8/2= 0.9m

Step three

Substituting our given data we have

V= 4/3*3.142*0.9³

V= 4/3*3.142*0.729

V= 9.162/3

V=3.05m³

The table shows the elevation in feet at the peaks of several mountains.


Mountain Elevation (feet)
Mt. Aspen 20,199.5
Snow Crest 28,882.31
Mt. Bethune 22,934.71
Parker Peak 14,987.42


Snow Crest is 11,873.21 feet higher than Mt. Wilson. Write and solve an equation to find the elevation of Mt. Wilson. Let x represent the elevation of Mt. Wilson.


The equation to find the elevation of Mt. Wilson is - x =
The elevation of Mt. Wilson is feet.

Answers

Answer:

X = Y - 11873.21

X = 17009.1 feet

Step-by-step explanation:

Let x represent the elevation of Mt. Wilson. And

Y = Mt snow Crest.

Since Snow Crest is 11,873.21 feet higher than Mt. Wilson, the equation will be

X = Y - 11873.21

Where Y = 28,882.31

Substitute Y into the equation

X = 28882.31 - 11873.21

X = 17009.1 feet.

this table shows values represented by an exponential function. What is the average rate of change for the function for the interval from x=2 to x=4?

Answers

Answer:

B

Step-by-step explanation:

2= 9

4= 64

64-9=55

If you subtract that

You will get 55

Answer:

the correct answer is 27.5

Step-by-step explanation:

How many triangles exist with the given angle measures?

Answers

Answer:  An infinite number

We're given angles adding to 180 degrees, so we can generate as many similar triangles with these angles as we like.  They'll all have the same shape but different sizes.

Answer:

1

Step-by-step explanation:

Find the reference angle of 14 pie over 11

Answers

Answer:

Step-by-step explanation:

reference angle=(14 π)/11-π=(14-11)π/11=3π/11

Which of the following describe a simple event?

Answers

Answer: B

Step-by-step explanation:

if the relationship is proportional, what is the missing value from the table?
A 8
B 9
C 12
D 14​

Answers

Answer:

C 12

Step-by-step explanation:

16 × 6 ÷ 8 = 12

check answer: 24 × 12 ÷ 16 = 18

i have alot of work to dk sk if anyone could help pleas​

Answers

All in the attachment :)

and don't complain about my drawing skills ;)

Find the
volumes of the two rectangular prisms
described in the table. If you divide
each dimension of the larger prism
by 2, how does the new volume
compare to its original volume?
Explain.

Answers

Answer:

1. 135 2. 16.875

Step-by-step explanation:

1. 5 x 4.5 x 6 = 135

2. 2.5 x 2.25 x 3 = 16.875

14 subtracted from the difference of x and 3

Answers

Answer:

3 is the answer. lol lol

Answer:

3

Step-by-step explanation:

The difference of x and 3

Difference means subtract, so we subtract 3 from x

x-3

14 subtracted from

14 is subtracted from the difference of x and 3, so

(x-3)-14

Therefore, the correct choice is 3: (x-3)-14

"Five less than three times a number is greater than two-thirds of that number."
Which inequality matches the given statement?
A. 3n-5>2/3n
B. 5-3n>2/3n
C. 3n-5<2/3n
D. 5-3n<2/3n

Answers

Answer:

A.

Step-by-step explanation:

The inequality matches the given statement  3n-5>2/3n.

What is the inequality?

An inequality is a relationship between two different quantities or expressions.

An inequality may be expressed by a mathematical sentence that uses the following symbols:

< is less than

> is greater than

≤ is less than or equal to

≥ is greater than or equal to

≠ is b equal to

"Five less than three times a number is greater than two-thirds of that number.

3n-5>2/3n

Hence, the inequality matches the given statement  3n-5>2/3n.

Learn more about inequality here;

https://brainly.com/question/11968331

#SPJ2

Circle O has a circumference of approximately 28.3 cm. What is the approximate
length of the radius?

Answers

Answer:

4.5 cm

Step-by-step explanation:

The circumference of a circle is given by

C = 2*pi*r where r is the radius

28.3 = 2 * (3.14) *r

28.3 = 6.28 r

Divide each side by 6.28

28.3/6.28 = r

4.506369427=r

The approximate length of the radius is 4.5 cm

which store will she purchase the rug from

Answers

store a: 10% of 45 is 4.5 45-4.5=40.5- can’t get this one with or without tax bc she only has $40 to spend
store b: 45-10=35 35+6% tax= 37.10- can afford with tax and would have $2.93 left over
the correct answer would be c

Which could be the entire interval over which the function, f(x), is negative?
(-8, -2)
(-8, 0)
(-infinity, -6)
(-infinity, -4)

Answers

The fourth answer is the true

The interval over which the function f(x) is negative is (-infinity, -4) .

Given,

The values of f(x) for corresponding x .

Here,

From the table,

x = -8 ; f(x) = -16

x = -6 ; f(x) = -8

Thus from the data given in the table it is clear that the function f(x) will be negative if we decrease the value of x .

Thus f(x) will be negative for the values which are smaller than -8 .

Now,

When

x = -4 ; f(x) = 0

x = -2 ; f(x) = 8

Thus f(x) changes its sign at x = -4.

So,

Interval in which f(x)  will be negative : (-infinity, -4) .

Know more about functions,

https://brainly.com/question/30845151

#SPJ2

For the most part the Supreme Court has ______ jurisdiction. The authority to see if a mistake was made in a case. ok so i accidentally put math great goin me

Answers

Answer:

Original

Step-by-step explanation:

what is the answer to 7+6y >19 ?

Answers

Answer:

[tex]y>2[/tex]

[tex]\{y\in \mathbb{R}|y>2\}[/tex]

The interval notation is: [tex](2, \infty)[/tex]

Step-by-step explanation:

[tex]7+6y>19[/tex]

[tex]6y>12[/tex]

[tex]y>2[/tex]

Which fraction is greater than 3/5

4/6

6/12

3/8

4/10

Answers

Answer:

a.) 4/6

Step-by-step explanation:

3/5 = .6

4/6 = .666666666667

6/12= 0.5

3/8= .375

4/10= .4

Answer: 4/6

Step-by-step explanation:

IT IS THE BIGGEST. :)

Evaluate the expression when x=2.3, y= -1.6, and z= -4.

x-y+z​

Answers

Answer:

-.01

Step-by-step explanation:

X - Y + Z

2.3 - (-1.6) + (-4)

2.3 + 1.6 - 4

-0.1

I hope this is helpful!!!!!!! :)

Answer:

-0.1

Hope this helped :)

I need help please. I WILL GIVE BRAINLY!!!!

Answers

Answer:

1.

The amount of time could be related to the battery charge by equation:

y = ax + b

a = 16% per 30 min = 32% per hour

b = 10%

=> y = 32x + 10

2.

In term of charging time, the rate of charging is 32% per hour

Which point could be on the line that is perpendicular to Line M N and passes through point K? (0, −12) (2, 2) (4, 8) (5, 13)

Answers

Answer:

B: (2,2)

Step-by-step explanation:

Answer:

B

Step-by-step explanation:

Find the number of positive numbers less than 2020, which can be written as the sum of five consecutive positive numbers.

Answers

Integers*?

Let [tex]n[/tex] be the smallest of these five consecutive integers. Then the sum of the five consecutive integers is [tex]\frac{5(2n+4)}{2}=5n+10,[/tex] so we seek the largest positive integer [tex]n[/tex] such that [tex]5n+10<2020.[/tex] Solving this inequality, we find that [tex]n<402,[/tex] so there are [tex]\boxed{401}[/tex] such positive integers.

Can someone explain and answer

Answers

The answer is D. y = -2x + 20. This is because it has a negative slope (-2x). When a number in a slope-intercept equation has the variable (x), that determines the positive or negative association. If it is a positive slope, it travels up from left to right. If it is a negative slope, it travels down from left to right.

Answer:

D. y = -2x + 20

Step-by-step explanation:

first, plot these points on a linear graph.

work down from the top:

A. y = 2x + 5

look at the last number first, this means the line will cross over the y axis (vertical line) at + 5. draw a point on + 5

y = 2x + 5

From this point (+5), the 2x means that for every 1 square you move across (to the right), you will go up 2. This will make the second point of the line. connect the dots. (see picture 1 below)

B. y = 2x - 6

the last number means the first point will be on -6 on the y axis.

y = 2x - 6

the 2x means that every 1 square you go across (to the right) from that point, you will move up 2

C. y = 2x

as there is no second number, this line will cross through the center of the graph. the 2 means that for every 1 square you move across from that point, you will move up 2

D. y = -2x + 20

the +20 means this line will cross the y axis and + 20, the -2 means that for every 1 square you move across to the right, you move DOWN 2.

this is the only line that moves down from left to right

Other Questions
30 POINTS PLEASE ANSWER CORRECTLY AND CLEARLY THANK YOU!Which expression represents the growth of an investment of $1,500 at 12% per year for a period of t years?A. 1500(12)tB. 1500(0.12)tC. 1500t1.12D. 1500(1.12)t 1. It is made up of white and red blood cells and platelets and carries oxygen to organs aroundthe body answer 2. This is developed by the body's defense system to destroy pathogens.Answer:3. Resistance to a particular disease.Answer:4. A wall-like, first line of defense against disease, illness and pathogens.einer5.it trash pathogens and foreign mucus and cilia 6.this is a part of a pathogen that signal the body immune system to respond what is the change in size of a group of organisms of the same species over time A) population growthB) death rate C) birth rateD) immigration and emigration b) Imagine that you earned $8,425 in one year. If the government enforces a 15% income tax, howmuch money would you owe in taxes at the end of the year? Show your work. (5 points) which one isnt a function ? On January 1, 2021, Legion Company sold $250,000 of 6% ten-year bonds. Interest is payable semiannually on June 30 and December 31. The bonds were sold for $163,976, priced to yield 12%. Legion records interest at the effective rate. Legion should report bond interest expense for the six months ended June 30, 2021, in the amount of: (Round your answer to the nearest dollar amount.) Rina is 6x + 3:8 feet tall and Ryan is 5x + 1/8 feet tall. How much taller is Rina than Ryan? what type of art has no visual subject But if he is suffered to live, he will receive the worst punishment - obscurity - and the public will have nothing to admire. We also know, and we can not consider it too much, that he has killed no man, and that if he be put to death we shall have the anomaly [odd situation] of the victim surviving the murderer; and that, under the laws, this man can be punished only for assault and battery with intent to kill, and, therefore, imprisoned.Background: This passage is an excerpt from William E. Dosters defense of Lewis Powell.According to the passage, what does Doster suggest that Powells punishment should be?becoming anonymous to the public becoming a hero and admiredbecoming a prisonerbecoming a victim of murder Given the following graph...A. Evaluate f(4)B. Solve for f(x)=1 How are businesses affected by globalization? a)Quality employees become hard to find. b)Companies are founded at a faster rate. c)Profits decrease due to increased competition. d)Goods are exchanged across the world at a rapid pace. If you answer can you write how you solved it? How did the policy of Fordism affect workers?O A. They had no options for health care.O B. They gained higher wages and could buy more goods.O C. They lost pensions when companies refused to pay.O D. They were forced to join labor unions. IL GIVE YOU BRAINLIST !! * have to get it right * The slope and y-intercept for a linear equation are given. Which graph matches this information? It is not a mans duty, as a matter of course, to devote himself to the eradication of any, even the most enormous, wrong; he may still properly have other concerns to engage him; but it is his duty, at least, to wash his hands of it, and, if he gives it no thought longer, not to give it practically his support. If I devote myself to other pursuits and contemplations, I must first see, at least, that I do not pursue them sitting upon another mans shoulders. I must get off him first, that he may pursue his contemplations too. See what gross inconsistency is tolerated. I have heard some of my townsmen say, I should like to have them order me out to help put down an insurrection of the slaves, or to march to Mexico; see if I would go; and yet these very men have each, directly by their allegiance, and so indirectly, at least, by their money, furnished1 a substitute. what is the significance of the use of the word deliberately Why were there not more wars in the North during the civil war? What is the surface area? I got 136 I needed someone to check it since Im not sure Sodium ions are used by your body every time you move a muscle. Determine the difference between the sodium ion and the sodium atom by using the model below. What is the volume, in cubic m, of a cube with an edge length of 18m?